Find $lim_{nto infty}((n+1)!)^{frac{1}{n+1}}-((n)!)^{frac{1}{n}}.$ [duplicate]












4












$begingroup$



This question already has an answer here:




  • Limit of the sequence $a_n=sqrt[n+1]{(n+1)!}-sqrt[n]{n!}$

    1 answer





Find $lim_{nto
infty}((n+1)!)^{frac{1}{n+1}}-((n)!)^{frac{1}{n}}.$




We need to deal the limit $lim_{nto infty} frac{log(1)+log(2)+...+log(n)}{n}$. We know that $lim_{nto infty} log(n)=infty implies lim_{nto infty} frac{log(1)+log(2)+...+log(n)}{n}=infty$(since, By Cauchy's first theorem on limit). Hence we get $infty-infty$. How do I show that there exists finite limit?










share|cite|improve this question











$endgroup$



marked as duplicate by Paramanand Singh limits
Users with the  limits badge can single-handedly close limits questions as duplicates and reopen them as needed.

StackExchange.ready(function() {
if (StackExchange.options.isMobile) return;

$('.dupe-hammer-message-hover:not(.hover-bound)').each(function() {
var $hover = $(this).addClass('hover-bound'),
$msg = $hover.siblings('.dupe-hammer-message');

$hover.hover(
function() {
$hover.showInfoMessage('', {
messageElement: $msg.clone().show(),
transient: false,
position: { my: 'bottom left', at: 'top center', offsetTop: -7 },
dismissable: false,
relativeToBody: true
});
},
function() {
StackExchange.helpers.removeMessages();
}
);
});
});
Dec 6 '18 at 16:10


This question has been asked before and already has an answer. If those answers do not fully address your question, please ask a new question.


















  • $begingroup$
    Let me check. But the limit is infinity.
    $endgroup$
    – Unknown x
    Dec 6 '18 at 16:01






  • 3




    $begingroup$
    The limit should be $1/e$.
    $endgroup$
    – Paramanand Singh
    Dec 6 '18 at 16:04










  • $begingroup$
    How do we get?Can you give some hint?
    $endgroup$
    – Unknown x
    Dec 6 '18 at 16:05
















4












$begingroup$



This question already has an answer here:




  • Limit of the sequence $a_n=sqrt[n+1]{(n+1)!}-sqrt[n]{n!}$

    1 answer





Find $lim_{nto
infty}((n+1)!)^{frac{1}{n+1}}-((n)!)^{frac{1}{n}}.$




We need to deal the limit $lim_{nto infty} frac{log(1)+log(2)+...+log(n)}{n}$. We know that $lim_{nto infty} log(n)=infty implies lim_{nto infty} frac{log(1)+log(2)+...+log(n)}{n}=infty$(since, By Cauchy's first theorem on limit). Hence we get $infty-infty$. How do I show that there exists finite limit?










share|cite|improve this question











$endgroup$



marked as duplicate by Paramanand Singh limits
Users with the  limits badge can single-handedly close limits questions as duplicates and reopen them as needed.

StackExchange.ready(function() {
if (StackExchange.options.isMobile) return;

$('.dupe-hammer-message-hover:not(.hover-bound)').each(function() {
var $hover = $(this).addClass('hover-bound'),
$msg = $hover.siblings('.dupe-hammer-message');

$hover.hover(
function() {
$hover.showInfoMessage('', {
messageElement: $msg.clone().show(),
transient: false,
position: { my: 'bottom left', at: 'top center', offsetTop: -7 },
dismissable: false,
relativeToBody: true
});
},
function() {
StackExchange.helpers.removeMessages();
}
);
});
});
Dec 6 '18 at 16:10


This question has been asked before and already has an answer. If those answers do not fully address your question, please ask a new question.


















  • $begingroup$
    Let me check. But the limit is infinity.
    $endgroup$
    – Unknown x
    Dec 6 '18 at 16:01






  • 3




    $begingroup$
    The limit should be $1/e$.
    $endgroup$
    – Paramanand Singh
    Dec 6 '18 at 16:04










  • $begingroup$
    How do we get?Can you give some hint?
    $endgroup$
    – Unknown x
    Dec 6 '18 at 16:05














4












4








4


1



$begingroup$



This question already has an answer here:




  • Limit of the sequence $a_n=sqrt[n+1]{(n+1)!}-sqrt[n]{n!}$

    1 answer





Find $lim_{nto
infty}((n+1)!)^{frac{1}{n+1}}-((n)!)^{frac{1}{n}}.$




We need to deal the limit $lim_{nto infty} frac{log(1)+log(2)+...+log(n)}{n}$. We know that $lim_{nto infty} log(n)=infty implies lim_{nto infty} frac{log(1)+log(2)+...+log(n)}{n}=infty$(since, By Cauchy's first theorem on limit). Hence we get $infty-infty$. How do I show that there exists finite limit?










share|cite|improve this question











$endgroup$





This question already has an answer here:




  • Limit of the sequence $a_n=sqrt[n+1]{(n+1)!}-sqrt[n]{n!}$

    1 answer





Find $lim_{nto
infty}((n+1)!)^{frac{1}{n+1}}-((n)!)^{frac{1}{n}}.$




We need to deal the limit $lim_{nto infty} frac{log(1)+log(2)+...+log(n)}{n}$. We know that $lim_{nto infty} log(n)=infty implies lim_{nto infty} frac{log(1)+log(2)+...+log(n)}{n}=infty$(since, By Cauchy's first theorem on limit). Hence we get $infty-infty$. How do I show that there exists finite limit?





This question already has an answer here:




  • Limit of the sequence $a_n=sqrt[n+1]{(n+1)!}-sqrt[n]{n!}$

    1 answer








real-analysis sequences-and-series limits






share|cite|improve this question















share|cite|improve this question













share|cite|improve this question




share|cite|improve this question








edited Dec 6 '18 at 15:59







Unknown x

















asked Dec 6 '18 at 15:57









Unknown xUnknown x

2,51211026




2,51211026




marked as duplicate by Paramanand Singh limits
Users with the  limits badge can single-handedly close limits questions as duplicates and reopen them as needed.

StackExchange.ready(function() {
if (StackExchange.options.isMobile) return;

$('.dupe-hammer-message-hover:not(.hover-bound)').each(function() {
var $hover = $(this).addClass('hover-bound'),
$msg = $hover.siblings('.dupe-hammer-message');

$hover.hover(
function() {
$hover.showInfoMessage('', {
messageElement: $msg.clone().show(),
transient: false,
position: { my: 'bottom left', at: 'top center', offsetTop: -7 },
dismissable: false,
relativeToBody: true
});
},
function() {
StackExchange.helpers.removeMessages();
}
);
});
});
Dec 6 '18 at 16:10


This question has been asked before and already has an answer. If those answers do not fully address your question, please ask a new question.









marked as duplicate by Paramanand Singh limits
Users with the  limits badge can single-handedly close limits questions as duplicates and reopen them as needed.

StackExchange.ready(function() {
if (StackExchange.options.isMobile) return;

$('.dupe-hammer-message-hover:not(.hover-bound)').each(function() {
var $hover = $(this).addClass('hover-bound'),
$msg = $hover.siblings('.dupe-hammer-message');

$hover.hover(
function() {
$hover.showInfoMessage('', {
messageElement: $msg.clone().show(),
transient: false,
position: { my: 'bottom left', at: 'top center', offsetTop: -7 },
dismissable: false,
relativeToBody: true
});
},
function() {
StackExchange.helpers.removeMessages();
}
);
});
});
Dec 6 '18 at 16:10


This question has been asked before and already has an answer. If those answers do not fully address your question, please ask a new question.














  • $begingroup$
    Let me check. But the limit is infinity.
    $endgroup$
    – Unknown x
    Dec 6 '18 at 16:01






  • 3




    $begingroup$
    The limit should be $1/e$.
    $endgroup$
    – Paramanand Singh
    Dec 6 '18 at 16:04










  • $begingroup$
    How do we get?Can you give some hint?
    $endgroup$
    – Unknown x
    Dec 6 '18 at 16:05


















  • $begingroup$
    Let me check. But the limit is infinity.
    $endgroup$
    – Unknown x
    Dec 6 '18 at 16:01






  • 3




    $begingroup$
    The limit should be $1/e$.
    $endgroup$
    – Paramanand Singh
    Dec 6 '18 at 16:04










  • $begingroup$
    How do we get?Can you give some hint?
    $endgroup$
    – Unknown x
    Dec 6 '18 at 16:05
















$begingroup$
Let me check. But the limit is infinity.
$endgroup$
– Unknown x
Dec 6 '18 at 16:01




$begingroup$
Let me check. But the limit is infinity.
$endgroup$
– Unknown x
Dec 6 '18 at 16:01




3




3




$begingroup$
The limit should be $1/e$.
$endgroup$
– Paramanand Singh
Dec 6 '18 at 16:04




$begingroup$
The limit should be $1/e$.
$endgroup$
– Paramanand Singh
Dec 6 '18 at 16:04












$begingroup$
How do we get?Can you give some hint?
$endgroup$
– Unknown x
Dec 6 '18 at 16:05




$begingroup$
How do we get?Can you give some hint?
$endgroup$
– Unknown x
Dec 6 '18 at 16:05










1 Answer
1






active

oldest

votes


















2












$begingroup$

HINT:



Using Stirling's Formula we have



$$begin{align}
left((n+1)!right)^{1/(n+1)}-left(n!right)^{1/n}&=left(left(sqrt{2pi(n+1)}left(frac{n+1}{e}right)^{n+1}right)left(1+O(1/n)right)right)^{1/(n+1)}\\
&-left(left(sqrt{2pi n}left(frac{n}{e}right)^{n}right)left(1+O(1/n)right)right)^{1/n}
end{align}$$



Can you finish now?






Alternatively, I provided a different, less "brute force" approach in THIS ANSWER.







share|cite|improve this answer











$endgroup$













  • $begingroup$
    This it is not convincing at all. Did you really check that this gets somewhere ?
    $endgroup$
    – Ewan Delanoy
    Dec 6 '18 at 16:13








  • 1




    $begingroup$
    @EwanDelanoy First of all, it is a HINT only. And yes, I did check this and found that the limit is simply $1/e$.
    $endgroup$
    – Mark Viola
    Dec 6 '18 at 16:15










  • $begingroup$
    @EwanDelanoy Alternatively, I provided a different, less "brute force" approach in THIS ANSWER.
    $endgroup$
    – Mark Viola
    Dec 6 '18 at 16:22










  • $begingroup$
    Is your sirling formula correct? $sqrt{2 pi n}$ in the numerator?
    $endgroup$
    – Unknown x
    Dec 6 '18 at 17:23










  • $begingroup$
    your alternate approach is very excellent :)
    $endgroup$
    – Unknown x
    Dec 6 '18 at 17:25


















1 Answer
1






active

oldest

votes








1 Answer
1






active

oldest

votes









active

oldest

votes






active

oldest

votes









2












$begingroup$

HINT:



Using Stirling's Formula we have



$$begin{align}
left((n+1)!right)^{1/(n+1)}-left(n!right)^{1/n}&=left(left(sqrt{2pi(n+1)}left(frac{n+1}{e}right)^{n+1}right)left(1+O(1/n)right)right)^{1/(n+1)}\\
&-left(left(sqrt{2pi n}left(frac{n}{e}right)^{n}right)left(1+O(1/n)right)right)^{1/n}
end{align}$$



Can you finish now?






Alternatively, I provided a different, less "brute force" approach in THIS ANSWER.







share|cite|improve this answer











$endgroup$













  • $begingroup$
    This it is not convincing at all. Did you really check that this gets somewhere ?
    $endgroup$
    – Ewan Delanoy
    Dec 6 '18 at 16:13








  • 1




    $begingroup$
    @EwanDelanoy First of all, it is a HINT only. And yes, I did check this and found that the limit is simply $1/e$.
    $endgroup$
    – Mark Viola
    Dec 6 '18 at 16:15










  • $begingroup$
    @EwanDelanoy Alternatively, I provided a different, less "brute force" approach in THIS ANSWER.
    $endgroup$
    – Mark Viola
    Dec 6 '18 at 16:22










  • $begingroup$
    Is your sirling formula correct? $sqrt{2 pi n}$ in the numerator?
    $endgroup$
    – Unknown x
    Dec 6 '18 at 17:23










  • $begingroup$
    your alternate approach is very excellent :)
    $endgroup$
    – Unknown x
    Dec 6 '18 at 17:25
















2












$begingroup$

HINT:



Using Stirling's Formula we have



$$begin{align}
left((n+1)!right)^{1/(n+1)}-left(n!right)^{1/n}&=left(left(sqrt{2pi(n+1)}left(frac{n+1}{e}right)^{n+1}right)left(1+O(1/n)right)right)^{1/(n+1)}\\
&-left(left(sqrt{2pi n}left(frac{n}{e}right)^{n}right)left(1+O(1/n)right)right)^{1/n}
end{align}$$



Can you finish now?






Alternatively, I provided a different, less "brute force" approach in THIS ANSWER.







share|cite|improve this answer











$endgroup$













  • $begingroup$
    This it is not convincing at all. Did you really check that this gets somewhere ?
    $endgroup$
    – Ewan Delanoy
    Dec 6 '18 at 16:13








  • 1




    $begingroup$
    @EwanDelanoy First of all, it is a HINT only. And yes, I did check this and found that the limit is simply $1/e$.
    $endgroup$
    – Mark Viola
    Dec 6 '18 at 16:15










  • $begingroup$
    @EwanDelanoy Alternatively, I provided a different, less "brute force" approach in THIS ANSWER.
    $endgroup$
    – Mark Viola
    Dec 6 '18 at 16:22










  • $begingroup$
    Is your sirling formula correct? $sqrt{2 pi n}$ in the numerator?
    $endgroup$
    – Unknown x
    Dec 6 '18 at 17:23










  • $begingroup$
    your alternate approach is very excellent :)
    $endgroup$
    – Unknown x
    Dec 6 '18 at 17:25














2












2








2





$begingroup$

HINT:



Using Stirling's Formula we have



$$begin{align}
left((n+1)!right)^{1/(n+1)}-left(n!right)^{1/n}&=left(left(sqrt{2pi(n+1)}left(frac{n+1}{e}right)^{n+1}right)left(1+O(1/n)right)right)^{1/(n+1)}\\
&-left(left(sqrt{2pi n}left(frac{n}{e}right)^{n}right)left(1+O(1/n)right)right)^{1/n}
end{align}$$



Can you finish now?






Alternatively, I provided a different, less "brute force" approach in THIS ANSWER.







share|cite|improve this answer











$endgroup$



HINT:



Using Stirling's Formula we have



$$begin{align}
left((n+1)!right)^{1/(n+1)}-left(n!right)^{1/n}&=left(left(sqrt{2pi(n+1)}left(frac{n+1}{e}right)^{n+1}right)left(1+O(1/n)right)right)^{1/(n+1)}\\
&-left(left(sqrt{2pi n}left(frac{n}{e}right)^{n}right)left(1+O(1/n)right)right)^{1/n}
end{align}$$



Can you finish now?






Alternatively, I provided a different, less "brute force" approach in THIS ANSWER.








share|cite|improve this answer














share|cite|improve this answer



share|cite|improve this answer








edited Dec 6 '18 at 20:46

























answered Dec 6 '18 at 16:05









Mark ViolaMark Viola

132k1275173




132k1275173












  • $begingroup$
    This it is not convincing at all. Did you really check that this gets somewhere ?
    $endgroup$
    – Ewan Delanoy
    Dec 6 '18 at 16:13








  • 1




    $begingroup$
    @EwanDelanoy First of all, it is a HINT only. And yes, I did check this and found that the limit is simply $1/e$.
    $endgroup$
    – Mark Viola
    Dec 6 '18 at 16:15










  • $begingroup$
    @EwanDelanoy Alternatively, I provided a different, less "brute force" approach in THIS ANSWER.
    $endgroup$
    – Mark Viola
    Dec 6 '18 at 16:22










  • $begingroup$
    Is your sirling formula correct? $sqrt{2 pi n}$ in the numerator?
    $endgroup$
    – Unknown x
    Dec 6 '18 at 17:23










  • $begingroup$
    your alternate approach is very excellent :)
    $endgroup$
    – Unknown x
    Dec 6 '18 at 17:25


















  • $begingroup$
    This it is not convincing at all. Did you really check that this gets somewhere ?
    $endgroup$
    – Ewan Delanoy
    Dec 6 '18 at 16:13








  • 1




    $begingroup$
    @EwanDelanoy First of all, it is a HINT only. And yes, I did check this and found that the limit is simply $1/e$.
    $endgroup$
    – Mark Viola
    Dec 6 '18 at 16:15










  • $begingroup$
    @EwanDelanoy Alternatively, I provided a different, less "brute force" approach in THIS ANSWER.
    $endgroup$
    – Mark Viola
    Dec 6 '18 at 16:22










  • $begingroup$
    Is your sirling formula correct? $sqrt{2 pi n}$ in the numerator?
    $endgroup$
    – Unknown x
    Dec 6 '18 at 17:23










  • $begingroup$
    your alternate approach is very excellent :)
    $endgroup$
    – Unknown x
    Dec 6 '18 at 17:25
















$begingroup$
This it is not convincing at all. Did you really check that this gets somewhere ?
$endgroup$
– Ewan Delanoy
Dec 6 '18 at 16:13






$begingroup$
This it is not convincing at all. Did you really check that this gets somewhere ?
$endgroup$
– Ewan Delanoy
Dec 6 '18 at 16:13






1




1




$begingroup$
@EwanDelanoy First of all, it is a HINT only. And yes, I did check this and found that the limit is simply $1/e$.
$endgroup$
– Mark Viola
Dec 6 '18 at 16:15




$begingroup$
@EwanDelanoy First of all, it is a HINT only. And yes, I did check this and found that the limit is simply $1/e$.
$endgroup$
– Mark Viola
Dec 6 '18 at 16:15












$begingroup$
@EwanDelanoy Alternatively, I provided a different, less "brute force" approach in THIS ANSWER.
$endgroup$
– Mark Viola
Dec 6 '18 at 16:22




$begingroup$
@EwanDelanoy Alternatively, I provided a different, less "brute force" approach in THIS ANSWER.
$endgroup$
– Mark Viola
Dec 6 '18 at 16:22












$begingroup$
Is your sirling formula correct? $sqrt{2 pi n}$ in the numerator?
$endgroup$
– Unknown x
Dec 6 '18 at 17:23




$begingroup$
Is your sirling formula correct? $sqrt{2 pi n}$ in the numerator?
$endgroup$
– Unknown x
Dec 6 '18 at 17:23












$begingroup$
your alternate approach is very excellent :)
$endgroup$
– Unknown x
Dec 6 '18 at 17:25




$begingroup$
your alternate approach is very excellent :)
$endgroup$
– Unknown x
Dec 6 '18 at 17:25



Popular posts from this blog

Plaza Victoria

In PowerPoint, is there a keyboard shortcut for bulleted / numbered list?

How to put 3 figures in Latex with 2 figures side by side and 1 below these side by side images but in...